LSAT and Law School Admissions Forum

Get expert LSAT preparation and law school admissions advice from PowerScore Test Preparation.

User avatar
 Dave Killoran
PowerScore Staff
  • PowerScore Staff
  • Posts: 5852
  • Joined: Mar 25, 2011
|
#47076
Complete Question Explanation
(The complete setup for this game can be found here: lsat/viewtopic.php?t=8685)

The correct answer choice is (E)

There are a number of ways to do this question, including referring to the main diagram or by using hypotheticals.

Perhaps the easiest way to solve this question is to use the hypothetical from question #15. That solution shows that G3 and H3 could be empty, and only answer choice (E) contains both those seats. Thus, answer choice (E) is correct.

Get the most out of your LSAT Prep Plus subscription.

Analyze and track your performance with our Testing and Analytics Package.